Calculating an infinite sum: $sumlimits_{n=1}^{infty} 2frac{3^{n-1}n(n-1)}{(n+2)(n+1)!}$ [closed]












1












$begingroup$


I need to calculate the following formula



$$sum_{n=1}^{infty} 2frac{3^{n-1}n(n-1)}{(n+2)(n+1)!}$$



I know that this sum should be finite, but I don't know how to calculate it. I've done some simplifications and reductions but it get me nowhere. Please help me to calculate it.










share|cite|improve this question











$endgroup$



closed as off-topic by Simply Beautiful Art, José Carlos Santos, Holo, Did, RRL Jan 18 at 16:47


This question appears to be off-topic. The users who voted to close gave this specific reason:


  • "This question is missing context or other details: Please provide additional context, which ideally explains why the question is relevant to you and our community. Some forms of context include: background and motivation, relevant definitions, source, possible strategies, your current progress, why the question is interesting or important, etc." – Simply Beautiful Art, José Carlos Santos, Holo, Did, RRL

If this question can be reworded to fit the rules in the help center, please edit the question.





















    1












    $begingroup$


    I need to calculate the following formula



    $$sum_{n=1}^{infty} 2frac{3^{n-1}n(n-1)}{(n+2)(n+1)!}$$



    I know that this sum should be finite, but I don't know how to calculate it. I've done some simplifications and reductions but it get me nowhere. Please help me to calculate it.










    share|cite|improve this question











    $endgroup$



    closed as off-topic by Simply Beautiful Art, José Carlos Santos, Holo, Did, RRL Jan 18 at 16:47


    This question appears to be off-topic. The users who voted to close gave this specific reason:


    • "This question is missing context or other details: Please provide additional context, which ideally explains why the question is relevant to you and our community. Some forms of context include: background and motivation, relevant definitions, source, possible strategies, your current progress, why the question is interesting or important, etc." – Simply Beautiful Art, José Carlos Santos, Holo, Did, RRL

    If this question can be reworded to fit the rules in the help center, please edit the question.



















      1












      1








      1


      1



      $begingroup$


      I need to calculate the following formula



      $$sum_{n=1}^{infty} 2frac{3^{n-1}n(n-1)}{(n+2)(n+1)!}$$



      I know that this sum should be finite, but I don't know how to calculate it. I've done some simplifications and reductions but it get me nowhere. Please help me to calculate it.










      share|cite|improve this question











      $endgroup$




      I need to calculate the following formula



      $$sum_{n=1}^{infty} 2frac{3^{n-1}n(n-1)}{(n+2)(n+1)!}$$



      I know that this sum should be finite, but I don't know how to calculate it. I've done some simplifications and reductions but it get me nowhere. Please help me to calculate it.







      calculus sequences-and-series power-series






      share|cite|improve this question















      share|cite|improve this question













      share|cite|improve this question




      share|cite|improve this question








      edited Jan 18 at 15:20









      Martin Sleziak

      44.7k9117272




      44.7k9117272










      asked Jan 18 at 13:36









      Laina YabLaina Yab

      235




      235




      closed as off-topic by Simply Beautiful Art, José Carlos Santos, Holo, Did, RRL Jan 18 at 16:47


      This question appears to be off-topic. The users who voted to close gave this specific reason:


      • "This question is missing context or other details: Please provide additional context, which ideally explains why the question is relevant to you and our community. Some forms of context include: background and motivation, relevant definitions, source, possible strategies, your current progress, why the question is interesting or important, etc." – Simply Beautiful Art, José Carlos Santos, Holo, Did, RRL

      If this question can be reworded to fit the rules in the help center, please edit the question.







      closed as off-topic by Simply Beautiful Art, José Carlos Santos, Holo, Did, RRL Jan 18 at 16:47


      This question appears to be off-topic. The users who voted to close gave this specific reason:


      • "This question is missing context or other details: Please provide additional context, which ideally explains why the question is relevant to you and our community. Some forms of context include: background and motivation, relevant definitions, source, possible strategies, your current progress, why the question is interesting or important, etc." – Simply Beautiful Art, José Carlos Santos, Holo, Did, RRL

      If this question can be reworded to fit the rules in the help center, please edit the question.






















          2 Answers
          2






          active

          oldest

          votes


















          6












          $begingroup$

          You may proceed as follows:




          • $e^x = sum_{n=0}^{infty}frac{x^n}{n!} Rightarrow e^x -(1+x+frac{x^2}{2}) = sum_{color{blue}{n=3}}^{infty} frac{x^n}{n!} = sum_{color{blue}{n=1}}^{infty} frac{x^{n+2}}{(n+2)!}$

          • $Rightarrow f(x) = frac{e^x -(1+x+frac{x^2}{2})}{x^2} = sum_{n=1}^{infty} frac{x^{n}}{(n+2)!}$


          • $Rightarrow 2xf''(x) = boxed{2sum_{n=1}^{infty} frac{x^{n-1}n(n-1)}{(n+2)!}}$. This is your sum for $x=3$.

          • $Rightarrow 2xf''(x) = 2x frac{e^x(x^2-4x+6)-2(x+3)}{x^4}= 2frac{e^x(x^2-4x+6)-2(x+3)}{x^3}$

          • $x= 3 Rightarrow boxed{2sum_{n=1}^{infty} frac{3^{n-1}n(n-1)}{(n+2)!} =frac{2}{9}(e^3-4)}$






          share|cite|improve this answer









          $endgroup$









          • 1




            $begingroup$
            I couldn't resist saying this is very elegant
            $endgroup$
            – roman
            Jan 18 at 14:42



















          4












          $begingroup$

          Hint:



          Write $n(n-1)=(n+2)(n+1)+A(n+2)+B$



          $iff n^2-n=n^2+n(3+A)+2A+B+2$



          $3+A=-1iff A=-4,2A+B+2=0iff B=-2-2A=-2-2(-4)=6$



          Use $sum_{r=0}^inftydfrac{y^r}{r!}=e^y$



          $$dfrac{x^{n-1}n(n-1)}{(n+2)!}$$



          $$=x^{n-1}cdotdfrac{ (n+2)(n+1)-4(n+2)+6}{(n+2)!} =dfrac1xcdotdfrac{x^n}{n!}-dfrac4{x^2}cdotdfrac{x^{n+1}}{(n+1)!}+dfrac6{x^3}cdotdfrac{x^{n+2}}{(n+2)!}$$



          $$impliessum_{n=1}^inftydfrac{x^{n-1}n(n-1)}{(n+2)!}=dfrac1xsum_{n=1}^inftydfrac{x^n}{n!}-dfrac4{x^2}sum_{n=1}^inftydfrac{x^{n+1}}{(n+1)!}+dfrac6{x^3}sum_{n=1}^inftydfrac{x^{n+2}}{(n+2)!}$$



          $$=dfrac1xleft(e^x-1right)-dfrac4{x^2}left(e^x-1-dfrac x1right)+dfrac6{x^3}left(e^x-1-dfrac x1-dfrac{x^2}{2!}right)$$



          $$=e^xleft(dfrac1x-dfrac4{x^2}+dfrac6{x^3}right)+dfrac1xleft(-1+4-3right)+dfrac1{x^2}left(4+6right)-dfrac6{x^3}$$



          Here $x=3$






          share|cite|improve this answer











          $endgroup$













          • $begingroup$
            See also : math.stackexchange.com/questions/2638073/…
            $endgroup$
            – lab bhattacharjee
            Jan 19 at 4:02


















          2 Answers
          2






          active

          oldest

          votes








          2 Answers
          2






          active

          oldest

          votes









          active

          oldest

          votes






          active

          oldest

          votes









          6












          $begingroup$

          You may proceed as follows:




          • $e^x = sum_{n=0}^{infty}frac{x^n}{n!} Rightarrow e^x -(1+x+frac{x^2}{2}) = sum_{color{blue}{n=3}}^{infty} frac{x^n}{n!} = sum_{color{blue}{n=1}}^{infty} frac{x^{n+2}}{(n+2)!}$

          • $Rightarrow f(x) = frac{e^x -(1+x+frac{x^2}{2})}{x^2} = sum_{n=1}^{infty} frac{x^{n}}{(n+2)!}$


          • $Rightarrow 2xf''(x) = boxed{2sum_{n=1}^{infty} frac{x^{n-1}n(n-1)}{(n+2)!}}$. This is your sum for $x=3$.

          • $Rightarrow 2xf''(x) = 2x frac{e^x(x^2-4x+6)-2(x+3)}{x^4}= 2frac{e^x(x^2-4x+6)-2(x+3)}{x^3}$

          • $x= 3 Rightarrow boxed{2sum_{n=1}^{infty} frac{3^{n-1}n(n-1)}{(n+2)!} =frac{2}{9}(e^3-4)}$






          share|cite|improve this answer









          $endgroup$









          • 1




            $begingroup$
            I couldn't resist saying this is very elegant
            $endgroup$
            – roman
            Jan 18 at 14:42
















          6












          $begingroup$

          You may proceed as follows:




          • $e^x = sum_{n=0}^{infty}frac{x^n}{n!} Rightarrow e^x -(1+x+frac{x^2}{2}) = sum_{color{blue}{n=3}}^{infty} frac{x^n}{n!} = sum_{color{blue}{n=1}}^{infty} frac{x^{n+2}}{(n+2)!}$

          • $Rightarrow f(x) = frac{e^x -(1+x+frac{x^2}{2})}{x^2} = sum_{n=1}^{infty} frac{x^{n}}{(n+2)!}$


          • $Rightarrow 2xf''(x) = boxed{2sum_{n=1}^{infty} frac{x^{n-1}n(n-1)}{(n+2)!}}$. This is your sum for $x=3$.

          • $Rightarrow 2xf''(x) = 2x frac{e^x(x^2-4x+6)-2(x+3)}{x^4}= 2frac{e^x(x^2-4x+6)-2(x+3)}{x^3}$

          • $x= 3 Rightarrow boxed{2sum_{n=1}^{infty} frac{3^{n-1}n(n-1)}{(n+2)!} =frac{2}{9}(e^3-4)}$






          share|cite|improve this answer









          $endgroup$









          • 1




            $begingroup$
            I couldn't resist saying this is very elegant
            $endgroup$
            – roman
            Jan 18 at 14:42














          6












          6








          6





          $begingroup$

          You may proceed as follows:




          • $e^x = sum_{n=0}^{infty}frac{x^n}{n!} Rightarrow e^x -(1+x+frac{x^2}{2}) = sum_{color{blue}{n=3}}^{infty} frac{x^n}{n!} = sum_{color{blue}{n=1}}^{infty} frac{x^{n+2}}{(n+2)!}$

          • $Rightarrow f(x) = frac{e^x -(1+x+frac{x^2}{2})}{x^2} = sum_{n=1}^{infty} frac{x^{n}}{(n+2)!}$


          • $Rightarrow 2xf''(x) = boxed{2sum_{n=1}^{infty} frac{x^{n-1}n(n-1)}{(n+2)!}}$. This is your sum for $x=3$.

          • $Rightarrow 2xf''(x) = 2x frac{e^x(x^2-4x+6)-2(x+3)}{x^4}= 2frac{e^x(x^2-4x+6)-2(x+3)}{x^3}$

          • $x= 3 Rightarrow boxed{2sum_{n=1}^{infty} frac{3^{n-1}n(n-1)}{(n+2)!} =frac{2}{9}(e^3-4)}$






          share|cite|improve this answer









          $endgroup$



          You may proceed as follows:




          • $e^x = sum_{n=0}^{infty}frac{x^n}{n!} Rightarrow e^x -(1+x+frac{x^2}{2}) = sum_{color{blue}{n=3}}^{infty} frac{x^n}{n!} = sum_{color{blue}{n=1}}^{infty} frac{x^{n+2}}{(n+2)!}$

          • $Rightarrow f(x) = frac{e^x -(1+x+frac{x^2}{2})}{x^2} = sum_{n=1}^{infty} frac{x^{n}}{(n+2)!}$


          • $Rightarrow 2xf''(x) = boxed{2sum_{n=1}^{infty} frac{x^{n-1}n(n-1)}{(n+2)!}}$. This is your sum for $x=3$.

          • $Rightarrow 2xf''(x) = 2x frac{e^x(x^2-4x+6)-2(x+3)}{x^4}= 2frac{e^x(x^2-4x+6)-2(x+3)}{x^3}$

          • $x= 3 Rightarrow boxed{2sum_{n=1}^{infty} frac{3^{n-1}n(n-1)}{(n+2)!} =frac{2}{9}(e^3-4)}$







          share|cite|improve this answer












          share|cite|improve this answer



          share|cite|improve this answer










          answered Jan 18 at 14:12









          trancelocationtrancelocation

          10.5k1722




          10.5k1722








          • 1




            $begingroup$
            I couldn't resist saying this is very elegant
            $endgroup$
            – roman
            Jan 18 at 14:42














          • 1




            $begingroup$
            I couldn't resist saying this is very elegant
            $endgroup$
            – roman
            Jan 18 at 14:42








          1




          1




          $begingroup$
          I couldn't resist saying this is very elegant
          $endgroup$
          – roman
          Jan 18 at 14:42




          $begingroup$
          I couldn't resist saying this is very elegant
          $endgroup$
          – roman
          Jan 18 at 14:42











          4












          $begingroup$

          Hint:



          Write $n(n-1)=(n+2)(n+1)+A(n+2)+B$



          $iff n^2-n=n^2+n(3+A)+2A+B+2$



          $3+A=-1iff A=-4,2A+B+2=0iff B=-2-2A=-2-2(-4)=6$



          Use $sum_{r=0}^inftydfrac{y^r}{r!}=e^y$



          $$dfrac{x^{n-1}n(n-1)}{(n+2)!}$$



          $$=x^{n-1}cdotdfrac{ (n+2)(n+1)-4(n+2)+6}{(n+2)!} =dfrac1xcdotdfrac{x^n}{n!}-dfrac4{x^2}cdotdfrac{x^{n+1}}{(n+1)!}+dfrac6{x^3}cdotdfrac{x^{n+2}}{(n+2)!}$$



          $$impliessum_{n=1}^inftydfrac{x^{n-1}n(n-1)}{(n+2)!}=dfrac1xsum_{n=1}^inftydfrac{x^n}{n!}-dfrac4{x^2}sum_{n=1}^inftydfrac{x^{n+1}}{(n+1)!}+dfrac6{x^3}sum_{n=1}^inftydfrac{x^{n+2}}{(n+2)!}$$



          $$=dfrac1xleft(e^x-1right)-dfrac4{x^2}left(e^x-1-dfrac x1right)+dfrac6{x^3}left(e^x-1-dfrac x1-dfrac{x^2}{2!}right)$$



          $$=e^xleft(dfrac1x-dfrac4{x^2}+dfrac6{x^3}right)+dfrac1xleft(-1+4-3right)+dfrac1{x^2}left(4+6right)-dfrac6{x^3}$$



          Here $x=3$






          share|cite|improve this answer











          $endgroup$













          • $begingroup$
            See also : math.stackexchange.com/questions/2638073/…
            $endgroup$
            – lab bhattacharjee
            Jan 19 at 4:02
















          4












          $begingroup$

          Hint:



          Write $n(n-1)=(n+2)(n+1)+A(n+2)+B$



          $iff n^2-n=n^2+n(3+A)+2A+B+2$



          $3+A=-1iff A=-4,2A+B+2=0iff B=-2-2A=-2-2(-4)=6$



          Use $sum_{r=0}^inftydfrac{y^r}{r!}=e^y$



          $$dfrac{x^{n-1}n(n-1)}{(n+2)!}$$



          $$=x^{n-1}cdotdfrac{ (n+2)(n+1)-4(n+2)+6}{(n+2)!} =dfrac1xcdotdfrac{x^n}{n!}-dfrac4{x^2}cdotdfrac{x^{n+1}}{(n+1)!}+dfrac6{x^3}cdotdfrac{x^{n+2}}{(n+2)!}$$



          $$impliessum_{n=1}^inftydfrac{x^{n-1}n(n-1)}{(n+2)!}=dfrac1xsum_{n=1}^inftydfrac{x^n}{n!}-dfrac4{x^2}sum_{n=1}^inftydfrac{x^{n+1}}{(n+1)!}+dfrac6{x^3}sum_{n=1}^inftydfrac{x^{n+2}}{(n+2)!}$$



          $$=dfrac1xleft(e^x-1right)-dfrac4{x^2}left(e^x-1-dfrac x1right)+dfrac6{x^3}left(e^x-1-dfrac x1-dfrac{x^2}{2!}right)$$



          $$=e^xleft(dfrac1x-dfrac4{x^2}+dfrac6{x^3}right)+dfrac1xleft(-1+4-3right)+dfrac1{x^2}left(4+6right)-dfrac6{x^3}$$



          Here $x=3$






          share|cite|improve this answer











          $endgroup$













          • $begingroup$
            See also : math.stackexchange.com/questions/2638073/…
            $endgroup$
            – lab bhattacharjee
            Jan 19 at 4:02














          4












          4








          4





          $begingroup$

          Hint:



          Write $n(n-1)=(n+2)(n+1)+A(n+2)+B$



          $iff n^2-n=n^2+n(3+A)+2A+B+2$



          $3+A=-1iff A=-4,2A+B+2=0iff B=-2-2A=-2-2(-4)=6$



          Use $sum_{r=0}^inftydfrac{y^r}{r!}=e^y$



          $$dfrac{x^{n-1}n(n-1)}{(n+2)!}$$



          $$=x^{n-1}cdotdfrac{ (n+2)(n+1)-4(n+2)+6}{(n+2)!} =dfrac1xcdotdfrac{x^n}{n!}-dfrac4{x^2}cdotdfrac{x^{n+1}}{(n+1)!}+dfrac6{x^3}cdotdfrac{x^{n+2}}{(n+2)!}$$



          $$impliessum_{n=1}^inftydfrac{x^{n-1}n(n-1)}{(n+2)!}=dfrac1xsum_{n=1}^inftydfrac{x^n}{n!}-dfrac4{x^2}sum_{n=1}^inftydfrac{x^{n+1}}{(n+1)!}+dfrac6{x^3}sum_{n=1}^inftydfrac{x^{n+2}}{(n+2)!}$$



          $$=dfrac1xleft(e^x-1right)-dfrac4{x^2}left(e^x-1-dfrac x1right)+dfrac6{x^3}left(e^x-1-dfrac x1-dfrac{x^2}{2!}right)$$



          $$=e^xleft(dfrac1x-dfrac4{x^2}+dfrac6{x^3}right)+dfrac1xleft(-1+4-3right)+dfrac1{x^2}left(4+6right)-dfrac6{x^3}$$



          Here $x=3$






          share|cite|improve this answer











          $endgroup$



          Hint:



          Write $n(n-1)=(n+2)(n+1)+A(n+2)+B$



          $iff n^2-n=n^2+n(3+A)+2A+B+2$



          $3+A=-1iff A=-4,2A+B+2=0iff B=-2-2A=-2-2(-4)=6$



          Use $sum_{r=0}^inftydfrac{y^r}{r!}=e^y$



          $$dfrac{x^{n-1}n(n-1)}{(n+2)!}$$



          $$=x^{n-1}cdotdfrac{ (n+2)(n+1)-4(n+2)+6}{(n+2)!} =dfrac1xcdotdfrac{x^n}{n!}-dfrac4{x^2}cdotdfrac{x^{n+1}}{(n+1)!}+dfrac6{x^3}cdotdfrac{x^{n+2}}{(n+2)!}$$



          $$impliessum_{n=1}^inftydfrac{x^{n-1}n(n-1)}{(n+2)!}=dfrac1xsum_{n=1}^inftydfrac{x^n}{n!}-dfrac4{x^2}sum_{n=1}^inftydfrac{x^{n+1}}{(n+1)!}+dfrac6{x^3}sum_{n=1}^inftydfrac{x^{n+2}}{(n+2)!}$$



          $$=dfrac1xleft(e^x-1right)-dfrac4{x^2}left(e^x-1-dfrac x1right)+dfrac6{x^3}left(e^x-1-dfrac x1-dfrac{x^2}{2!}right)$$



          $$=e^xleft(dfrac1x-dfrac4{x^2}+dfrac6{x^3}right)+dfrac1xleft(-1+4-3right)+dfrac1{x^2}left(4+6right)-dfrac6{x^3}$$



          Here $x=3$







          share|cite|improve this answer














          share|cite|improve this answer



          share|cite|improve this answer








          edited Jan 18 at 16:23

























          answered Jan 18 at 13:48









          lab bhattacharjeelab bhattacharjee

          225k15156274




          225k15156274












          • $begingroup$
            See also : math.stackexchange.com/questions/2638073/…
            $endgroup$
            – lab bhattacharjee
            Jan 19 at 4:02


















          • $begingroup$
            See also : math.stackexchange.com/questions/2638073/…
            $endgroup$
            – lab bhattacharjee
            Jan 19 at 4:02
















          $begingroup$
          See also : math.stackexchange.com/questions/2638073/…
          $endgroup$
          – lab bhattacharjee
          Jan 19 at 4:02




          $begingroup$
          See also : math.stackexchange.com/questions/2638073/…
          $endgroup$
          – lab bhattacharjee
          Jan 19 at 4:02



          Popular posts from this blog

          Biblatex bibliography style without URLs when DOI exists (in Overleaf with Zotero bibliography)

          ComboBox Display Member on multiple fields

          Is it possible to collect Nectar points via Trainline?